$a^n-a + 1 $ divisible by $n$












19












$begingroup$



Problem. Given $a$ is a positive integer greater than 3, are there infinitely many positive integers $n$ satisfying $a^n-a + 1 $
divisible by $n$?











share|cite|improve this question











$endgroup$








  • 5




    $begingroup$
    Interesting question. Have you tried anything yet?
    $endgroup$
    – Mohammad Zuhair Khan
    Oct 24 '18 at 8:30






  • 3




    $begingroup$
    The case $a=2$ is popular, see here and the related links.
    $endgroup$
    – Dietrich Burde
    Oct 24 '18 at 8:48






  • 1




    $begingroup$
    $n$ is not a prime number. It also must be odd.
    $endgroup$
    – Oldboy
    Oct 24 '18 at 18:16






  • 2




    $begingroup$
    My original problem: Let $ainmathbb Z$ and $a>3$, prove that there exist infinitely many positive integers $n$ satisfying$$(n+a)mid left(a^n+1right).$$
    $endgroup$
    – Drona
    Oct 26 '18 at 19:54






  • 6




    $begingroup$
    @Drona I think that you are making a big mistake. The statement $(n+a)|(a^n+1)$ is not equivalent to $n|(a^n-a+1)$. It's like saying that $(4+3)|14$ is equivalent to $4|(14-3)$. You should post the original problem. If you don't want to do it, I would like to do it.
    $endgroup$
    – Oldboy
    Oct 27 '18 at 8:39


















19












$begingroup$



Problem. Given $a$ is a positive integer greater than 3, are there infinitely many positive integers $n$ satisfying $a^n-a + 1 $
divisible by $n$?











share|cite|improve this question











$endgroup$








  • 5




    $begingroup$
    Interesting question. Have you tried anything yet?
    $endgroup$
    – Mohammad Zuhair Khan
    Oct 24 '18 at 8:30






  • 3




    $begingroup$
    The case $a=2$ is popular, see here and the related links.
    $endgroup$
    – Dietrich Burde
    Oct 24 '18 at 8:48






  • 1




    $begingroup$
    $n$ is not a prime number. It also must be odd.
    $endgroup$
    – Oldboy
    Oct 24 '18 at 18:16






  • 2




    $begingroup$
    My original problem: Let $ainmathbb Z$ and $a>3$, prove that there exist infinitely many positive integers $n$ satisfying$$(n+a)mid left(a^n+1right).$$
    $endgroup$
    – Drona
    Oct 26 '18 at 19:54






  • 6




    $begingroup$
    @Drona I think that you are making a big mistake. The statement $(n+a)|(a^n+1)$ is not equivalent to $n|(a^n-a+1)$. It's like saying that $(4+3)|14$ is equivalent to $4|(14-3)$. You should post the original problem. If you don't want to do it, I would like to do it.
    $endgroup$
    – Oldboy
    Oct 27 '18 at 8:39
















19












19








19


11



$begingroup$



Problem. Given $a$ is a positive integer greater than 3, are there infinitely many positive integers $n$ satisfying $a^n-a + 1 $
divisible by $n$?











share|cite|improve this question











$endgroup$





Problem. Given $a$ is a positive integer greater than 3, are there infinitely many positive integers $n$ satisfying $a^n-a + 1 $
divisible by $n$?








number-theory elementary-number-theory divisibility






share|cite|improve this question















share|cite|improve this question













share|cite|improve this question




share|cite|improve this question








edited Dec 7 '18 at 16:57









Klangen

1,73711334




1,73711334










asked Oct 24 '18 at 8:10









DronaDrona

1627




1627








  • 5




    $begingroup$
    Interesting question. Have you tried anything yet?
    $endgroup$
    – Mohammad Zuhair Khan
    Oct 24 '18 at 8:30






  • 3




    $begingroup$
    The case $a=2$ is popular, see here and the related links.
    $endgroup$
    – Dietrich Burde
    Oct 24 '18 at 8:48






  • 1




    $begingroup$
    $n$ is not a prime number. It also must be odd.
    $endgroup$
    – Oldboy
    Oct 24 '18 at 18:16






  • 2




    $begingroup$
    My original problem: Let $ainmathbb Z$ and $a>3$, prove that there exist infinitely many positive integers $n$ satisfying$$(n+a)mid left(a^n+1right).$$
    $endgroup$
    – Drona
    Oct 26 '18 at 19:54






  • 6




    $begingroup$
    @Drona I think that you are making a big mistake. The statement $(n+a)|(a^n+1)$ is not equivalent to $n|(a^n-a+1)$. It's like saying that $(4+3)|14$ is equivalent to $4|(14-3)$. You should post the original problem. If you don't want to do it, I would like to do it.
    $endgroup$
    – Oldboy
    Oct 27 '18 at 8:39
















  • 5




    $begingroup$
    Interesting question. Have you tried anything yet?
    $endgroup$
    – Mohammad Zuhair Khan
    Oct 24 '18 at 8:30






  • 3




    $begingroup$
    The case $a=2$ is popular, see here and the related links.
    $endgroup$
    – Dietrich Burde
    Oct 24 '18 at 8:48






  • 1




    $begingroup$
    $n$ is not a prime number. It also must be odd.
    $endgroup$
    – Oldboy
    Oct 24 '18 at 18:16






  • 2




    $begingroup$
    My original problem: Let $ainmathbb Z$ and $a>3$, prove that there exist infinitely many positive integers $n$ satisfying$$(n+a)mid left(a^n+1right).$$
    $endgroup$
    – Drona
    Oct 26 '18 at 19:54






  • 6




    $begingroup$
    @Drona I think that you are making a big mistake. The statement $(n+a)|(a^n+1)$ is not equivalent to $n|(a^n-a+1)$. It's like saying that $(4+3)|14$ is equivalent to $4|(14-3)$. You should post the original problem. If you don't want to do it, I would like to do it.
    $endgroup$
    – Oldboy
    Oct 27 '18 at 8:39










5




5




$begingroup$
Interesting question. Have you tried anything yet?
$endgroup$
– Mohammad Zuhair Khan
Oct 24 '18 at 8:30




$begingroup$
Interesting question. Have you tried anything yet?
$endgroup$
– Mohammad Zuhair Khan
Oct 24 '18 at 8:30




3




3




$begingroup$
The case $a=2$ is popular, see here and the related links.
$endgroup$
– Dietrich Burde
Oct 24 '18 at 8:48




$begingroup$
The case $a=2$ is popular, see here and the related links.
$endgroup$
– Dietrich Burde
Oct 24 '18 at 8:48




1




1




$begingroup$
$n$ is not a prime number. It also must be odd.
$endgroup$
– Oldboy
Oct 24 '18 at 18:16




$begingroup$
$n$ is not a prime number. It also must be odd.
$endgroup$
– Oldboy
Oct 24 '18 at 18:16




2




2




$begingroup$
My original problem: Let $ainmathbb Z$ and $a>3$, prove that there exist infinitely many positive integers $n$ satisfying$$(n+a)mid left(a^n+1right).$$
$endgroup$
– Drona
Oct 26 '18 at 19:54




$begingroup$
My original problem: Let $ainmathbb Z$ and $a>3$, prove that there exist infinitely many positive integers $n$ satisfying$$(n+a)mid left(a^n+1right).$$
$endgroup$
– Drona
Oct 26 '18 at 19:54




6




6




$begingroup$
@Drona I think that you are making a big mistake. The statement $(n+a)|(a^n+1)$ is not equivalent to $n|(a^n-a+1)$. It's like saying that $(4+3)|14$ is equivalent to $4|(14-3)$. You should post the original problem. If you don't want to do it, I would like to do it.
$endgroup$
– Oldboy
Oct 27 '18 at 8:39






$begingroup$
@Drona I think that you are making a big mistake. The statement $(n+a)|(a^n+1)$ is not equivalent to $n|(a^n-a+1)$. It's like saying that $(4+3)|14$ is equivalent to $4|(14-3)$. You should post the original problem. If you don't want to do it, I would like to do it.
$endgroup$
– Oldboy
Oct 27 '18 at 8:39












1 Answer
1






active

oldest

votes


















1












$begingroup$

$N=a^n-a+1$



$a^{p_1} ≡ a mod p_1$



$a^{p_2 } ≡a mod p_2$



$(a^{p_1})^{p_2} ≡ a^{p_2} mod p_1 ≡(a mod p_2) mod p_1= k_1 p_1 + k_2 p_2 + a$



$a^{p_1p_2}=k_1 p_1 + k_2 p_2 +a$



$a^{p_1p_2}-a+1=k_1p_1 +k_2p_2 +1$



If $n=p_1p_2 | a^n-a+1$ then we must have:



$p_1p_2 | k_1p_1 + k_2p_2+1$



So we have following linear equation:



$k_1p_1 + k_2 p_2 = m p_1p_2-1$



For certain value of $p_1$ and $p_2$ and m,there can be infinitely many solutions for $k_1$ and $k_2$. For example:



with $p_1=5$, $p_2=7$ and $m=3$ the equation has one solution like $k_1=11$ and $k_2=7$ and all other solutions can be found by:



$k_1= 7 t + 11$ and $k_2= -5 t+7$.



Now in first step problem reduces to:



Find n so that there exist a common divisor between $n$ and $N=a^n-a+1$.



For example for $a=3$, $p_1=5$ and $p_2=7$ we have:



$3^{35}-3+1=105$ and $(35, 105)=5$



In second step we must find m, $k_1$ and $k_2$ for certain amount of $p_1$ and $p_2$ so that $(n, N)=n$.



Relation $a^{p_1p_2}=k_1 p_1 + k_2 p_2 +a$ shows that $a|k_1 p_1 + k_2 p_2 $; if $k_1=u+1$ and $k_2=v-1$, $p_1= a .b+1$ and $p_2=a.c +1$, i.e. $p_1≡1mod a$ and $p_2≡1mod a$, then:



$k_1p_1+k_2p_2= M(a)$



Or: $a | k_1p_1+k_2p_2$



We can see this in solution $n=409times 9831853$ for $a=6$; $409=48times 6 +1$ and $9831853=1638642 times 6 +1$



This can help us in choosing a and primes $p_1$ and $p_2$



I see no reason for the lack of more solutions.






share|cite|improve this answer









$endgroup$













    Your Answer





    StackExchange.ifUsing("editor", function () {
    return StackExchange.using("mathjaxEditing", function () {
    StackExchange.MarkdownEditor.creationCallbacks.add(function (editor, postfix) {
    StackExchange.mathjaxEditing.prepareWmdForMathJax(editor, postfix, [["$", "$"], ["\\(","\\)"]]);
    });
    });
    }, "mathjax-editing");

    StackExchange.ready(function() {
    var channelOptions = {
    tags: "".split(" "),
    id: "69"
    };
    initTagRenderer("".split(" "), "".split(" "), channelOptions);

    StackExchange.using("externalEditor", function() {
    // Have to fire editor after snippets, if snippets enabled
    if (StackExchange.settings.snippets.snippetsEnabled) {
    StackExchange.using("snippets", function() {
    createEditor();
    });
    }
    else {
    createEditor();
    }
    });

    function createEditor() {
    StackExchange.prepareEditor({
    heartbeatType: 'answer',
    autoActivateHeartbeat: false,
    convertImagesToLinks: true,
    noModals: true,
    showLowRepImageUploadWarning: true,
    reputationToPostImages: 10,
    bindNavPrevention: true,
    postfix: "",
    imageUploader: {
    brandingHtml: "Powered by u003ca class="icon-imgur-white" href="https://imgur.com/"u003eu003c/au003e",
    contentPolicyHtml: "User contributions licensed under u003ca href="https://creativecommons.org/licenses/by-sa/3.0/"u003ecc by-sa 3.0 with attribution requiredu003c/au003e u003ca href="https://stackoverflow.com/legal/content-policy"u003e(content policy)u003c/au003e",
    allowUrls: true
    },
    noCode: true, onDemand: true,
    discardSelector: ".discard-answer"
    ,immediatelyShowMarkdownHelp:true
    });


    }
    });














    draft saved

    draft discarded


















    StackExchange.ready(
    function () {
    StackExchange.openid.initPostLogin('.new-post-login', 'https%3a%2f%2fmath.stackexchange.com%2fquestions%2f2968838%2fan-a-1-divisible-by-n%23new-answer', 'question_page');
    }
    );

    Post as a guest















    Required, but never shown

























    1 Answer
    1






    active

    oldest

    votes








    1 Answer
    1






    active

    oldest

    votes









    active

    oldest

    votes






    active

    oldest

    votes









    1












    $begingroup$

    $N=a^n-a+1$



    $a^{p_1} ≡ a mod p_1$



    $a^{p_2 } ≡a mod p_2$



    $(a^{p_1})^{p_2} ≡ a^{p_2} mod p_1 ≡(a mod p_2) mod p_1= k_1 p_1 + k_2 p_2 + a$



    $a^{p_1p_2}=k_1 p_1 + k_2 p_2 +a$



    $a^{p_1p_2}-a+1=k_1p_1 +k_2p_2 +1$



    If $n=p_1p_2 | a^n-a+1$ then we must have:



    $p_1p_2 | k_1p_1 + k_2p_2+1$



    So we have following linear equation:



    $k_1p_1 + k_2 p_2 = m p_1p_2-1$



    For certain value of $p_1$ and $p_2$ and m,there can be infinitely many solutions for $k_1$ and $k_2$. For example:



    with $p_1=5$, $p_2=7$ and $m=3$ the equation has one solution like $k_1=11$ and $k_2=7$ and all other solutions can be found by:



    $k_1= 7 t + 11$ and $k_2= -5 t+7$.



    Now in first step problem reduces to:



    Find n so that there exist a common divisor between $n$ and $N=a^n-a+1$.



    For example for $a=3$, $p_1=5$ and $p_2=7$ we have:



    $3^{35}-3+1=105$ and $(35, 105)=5$



    In second step we must find m, $k_1$ and $k_2$ for certain amount of $p_1$ and $p_2$ so that $(n, N)=n$.



    Relation $a^{p_1p_2}=k_1 p_1 + k_2 p_2 +a$ shows that $a|k_1 p_1 + k_2 p_2 $; if $k_1=u+1$ and $k_2=v-1$, $p_1= a .b+1$ and $p_2=a.c +1$, i.e. $p_1≡1mod a$ and $p_2≡1mod a$, then:



    $k_1p_1+k_2p_2= M(a)$



    Or: $a | k_1p_1+k_2p_2$



    We can see this in solution $n=409times 9831853$ for $a=6$; $409=48times 6 +1$ and $9831853=1638642 times 6 +1$



    This can help us in choosing a and primes $p_1$ and $p_2$



    I see no reason for the lack of more solutions.






    share|cite|improve this answer









    $endgroup$


















      1












      $begingroup$

      $N=a^n-a+1$



      $a^{p_1} ≡ a mod p_1$



      $a^{p_2 } ≡a mod p_2$



      $(a^{p_1})^{p_2} ≡ a^{p_2} mod p_1 ≡(a mod p_2) mod p_1= k_1 p_1 + k_2 p_2 + a$



      $a^{p_1p_2}=k_1 p_1 + k_2 p_2 +a$



      $a^{p_1p_2}-a+1=k_1p_1 +k_2p_2 +1$



      If $n=p_1p_2 | a^n-a+1$ then we must have:



      $p_1p_2 | k_1p_1 + k_2p_2+1$



      So we have following linear equation:



      $k_1p_1 + k_2 p_2 = m p_1p_2-1$



      For certain value of $p_1$ and $p_2$ and m,there can be infinitely many solutions for $k_1$ and $k_2$. For example:



      with $p_1=5$, $p_2=7$ and $m=3$ the equation has one solution like $k_1=11$ and $k_2=7$ and all other solutions can be found by:



      $k_1= 7 t + 11$ and $k_2= -5 t+7$.



      Now in first step problem reduces to:



      Find n so that there exist a common divisor between $n$ and $N=a^n-a+1$.



      For example for $a=3$, $p_1=5$ and $p_2=7$ we have:



      $3^{35}-3+1=105$ and $(35, 105)=5$



      In second step we must find m, $k_1$ and $k_2$ for certain amount of $p_1$ and $p_2$ so that $(n, N)=n$.



      Relation $a^{p_1p_2}=k_1 p_1 + k_2 p_2 +a$ shows that $a|k_1 p_1 + k_2 p_2 $; if $k_1=u+1$ and $k_2=v-1$, $p_1= a .b+1$ and $p_2=a.c +1$, i.e. $p_1≡1mod a$ and $p_2≡1mod a$, then:



      $k_1p_1+k_2p_2= M(a)$



      Or: $a | k_1p_1+k_2p_2$



      We can see this in solution $n=409times 9831853$ for $a=6$; $409=48times 6 +1$ and $9831853=1638642 times 6 +1$



      This can help us in choosing a and primes $p_1$ and $p_2$



      I see no reason for the lack of more solutions.






      share|cite|improve this answer









      $endgroup$
















        1












        1








        1





        $begingroup$

        $N=a^n-a+1$



        $a^{p_1} ≡ a mod p_1$



        $a^{p_2 } ≡a mod p_2$



        $(a^{p_1})^{p_2} ≡ a^{p_2} mod p_1 ≡(a mod p_2) mod p_1= k_1 p_1 + k_2 p_2 + a$



        $a^{p_1p_2}=k_1 p_1 + k_2 p_2 +a$



        $a^{p_1p_2}-a+1=k_1p_1 +k_2p_2 +1$



        If $n=p_1p_2 | a^n-a+1$ then we must have:



        $p_1p_2 | k_1p_1 + k_2p_2+1$



        So we have following linear equation:



        $k_1p_1 + k_2 p_2 = m p_1p_2-1$



        For certain value of $p_1$ and $p_2$ and m,there can be infinitely many solutions for $k_1$ and $k_2$. For example:



        with $p_1=5$, $p_2=7$ and $m=3$ the equation has one solution like $k_1=11$ and $k_2=7$ and all other solutions can be found by:



        $k_1= 7 t + 11$ and $k_2= -5 t+7$.



        Now in first step problem reduces to:



        Find n so that there exist a common divisor between $n$ and $N=a^n-a+1$.



        For example for $a=3$, $p_1=5$ and $p_2=7$ we have:



        $3^{35}-3+1=105$ and $(35, 105)=5$



        In second step we must find m, $k_1$ and $k_2$ for certain amount of $p_1$ and $p_2$ so that $(n, N)=n$.



        Relation $a^{p_1p_2}=k_1 p_1 + k_2 p_2 +a$ shows that $a|k_1 p_1 + k_2 p_2 $; if $k_1=u+1$ and $k_2=v-1$, $p_1= a .b+1$ and $p_2=a.c +1$, i.e. $p_1≡1mod a$ and $p_2≡1mod a$, then:



        $k_1p_1+k_2p_2= M(a)$



        Or: $a | k_1p_1+k_2p_2$



        We can see this in solution $n=409times 9831853$ for $a=6$; $409=48times 6 +1$ and $9831853=1638642 times 6 +1$



        This can help us in choosing a and primes $p_1$ and $p_2$



        I see no reason for the lack of more solutions.






        share|cite|improve this answer









        $endgroup$



        $N=a^n-a+1$



        $a^{p_1} ≡ a mod p_1$



        $a^{p_2 } ≡a mod p_2$



        $(a^{p_1})^{p_2} ≡ a^{p_2} mod p_1 ≡(a mod p_2) mod p_1= k_1 p_1 + k_2 p_2 + a$



        $a^{p_1p_2}=k_1 p_1 + k_2 p_2 +a$



        $a^{p_1p_2}-a+1=k_1p_1 +k_2p_2 +1$



        If $n=p_1p_2 | a^n-a+1$ then we must have:



        $p_1p_2 | k_1p_1 + k_2p_2+1$



        So we have following linear equation:



        $k_1p_1 + k_2 p_2 = m p_1p_2-1$



        For certain value of $p_1$ and $p_2$ and m,there can be infinitely many solutions for $k_1$ and $k_2$. For example:



        with $p_1=5$, $p_2=7$ and $m=3$ the equation has one solution like $k_1=11$ and $k_2=7$ and all other solutions can be found by:



        $k_1= 7 t + 11$ and $k_2= -5 t+7$.



        Now in first step problem reduces to:



        Find n so that there exist a common divisor between $n$ and $N=a^n-a+1$.



        For example for $a=3$, $p_1=5$ and $p_2=7$ we have:



        $3^{35}-3+1=105$ and $(35, 105)=5$



        In second step we must find m, $k_1$ and $k_2$ for certain amount of $p_1$ and $p_2$ so that $(n, N)=n$.



        Relation $a^{p_1p_2}=k_1 p_1 + k_2 p_2 +a$ shows that $a|k_1 p_1 + k_2 p_2 $; if $k_1=u+1$ and $k_2=v-1$, $p_1= a .b+1$ and $p_2=a.c +1$, i.e. $p_1≡1mod a$ and $p_2≡1mod a$, then:



        $k_1p_1+k_2p_2= M(a)$



        Or: $a | k_1p_1+k_2p_2$



        We can see this in solution $n=409times 9831853$ for $a=6$; $409=48times 6 +1$ and $9831853=1638642 times 6 +1$



        This can help us in choosing a and primes $p_1$ and $p_2$



        I see no reason for the lack of more solutions.







        share|cite|improve this answer












        share|cite|improve this answer



        share|cite|improve this answer










        answered Feb 13 at 9:19









        siroussirous

        1,6851514




        1,6851514






























            draft saved

            draft discarded




















































            Thanks for contributing an answer to Mathematics Stack Exchange!


            • Please be sure to answer the question. Provide details and share your research!

            But avoid



            • Asking for help, clarification, or responding to other answers.

            • Making statements based on opinion; back them up with references or personal experience.


            Use MathJax to format equations. MathJax reference.


            To learn more, see our tips on writing great answers.




            draft saved


            draft discarded














            StackExchange.ready(
            function () {
            StackExchange.openid.initPostLogin('.new-post-login', 'https%3a%2f%2fmath.stackexchange.com%2fquestions%2f2968838%2fan-a-1-divisible-by-n%23new-answer', 'question_page');
            }
            );

            Post as a guest















            Required, but never shown





















































            Required, but never shown














            Required, but never shown












            Required, but never shown







            Required, but never shown

































            Required, but never shown














            Required, but never shown












            Required, but never shown







            Required, but never shown







            Popular posts from this blog

            Biblatex bibliography style without URLs when DOI exists (in Overleaf with Zotero bibliography)

            ComboBox Display Member on multiple fields

            Is it possible to collect Nectar points via Trainline?